0
$\begingroup$

Let $k$ be an arbitrary base field and $K, L, M$ some fields over $k$ contained in a fixed overfield $\Omega$.

Question: Are there some "reasonable" assumptions (ie beyond a bunch of really "boring" assumptions) on $K, L, M$ which imply that

$$ K(M\cap L)=MK\cap LK $$

holds? Maybe something with linear disjointness? Concrete conjecture( see for motivation below why this guess): If $L$ and $K$ linearly disjoint to $M$? If yes, how to see in that case the inclusion $K(M\cap L)\supset MK\cap LK$?

Original motivation leading me to this problem: Even thought this looks elementary I still haven't found a satisfying argument why following equality holds: assume $k \subset K \subset L$ an arbitrary tower of not neccessary finite algebraic extensions of $k$ and $M=k(t)$ the field of Laurent series. Then I expect that it should hold

$$ (k(t) \cdot K) \cap L = K $$

independently how the involved fields are embedded in overfield $\Omega$, but I still not see how to deduce the inclusion $ (k(t) \cdot K) \cap L \subset K $, even thought that it looks "plausible". Does somebody see a quick trick how to seee it?

Since in this special situation $L=LK$ and $M$ linearly disjoint to $L$, we clearly have $L \cap M =k$, this can be translated to the problem above.

I noticed that here was posted identical question, with primary focus on number fields, but unfortunarely not received any answer.

#UPDATE: As @Wojowu pointed out posing certain assumtions on linear disjointness relations between the involved fields not give the statement. What about if we pose the "very" restrictive assumtion that $K \subset L$ algebraic over $k$ and $M$ purely transcendental over $k$, like in "original motivation" part? (Not sure where it can be really weakened)

$\endgroup$
3
  • 1
    $\begingroup$ The conjecture fails even for algebraic extensions with linear disjointness assumption - take $K=\Bbb Q(\sqrt{2}),L=\Bbb Q(\sqrt{3}),M=\Bbb Q(\sqrt{6})$. Then $MK=LK=\Bbb Q(\sqrt{2},\sqrt{3})$, but $K(M\cap L)=K$. $\endgroup$ Commented Nov 23, 2023 at 11:51
  • $\begingroup$ @Wojowu: right, so looks not so promising in case all $K,L,M$ were algebraic ext'ns. If one were to lower expectations even further orientating on the "original motivation"-paragraph, is it at least true if say $K \subset L$ algebraic over $k$, and $M$ purely transcendental? $\endgroup$ Commented Nov 23, 2023 at 15:51
  • 1
    $\begingroup$ Regarding "Original motivation": $k((t))\cdot K\subseteq K((t))$, and $K((t))/K$ is regular, hence linearly disjoint from $L/K$ $\endgroup$ Commented Nov 23, 2023 at 16:49

0

You must log in to answer this question.

Start asking to get answers

Find the answer to your question by asking.

Ask question

Explore related questions

See similar questions with these tags.